LSAT and Law School Admissions Forum

Get expert LSAT preparation and law school admissions advice from PowerScore Test Preparation.

 Administrator
PowerScore Staff
  • PowerScore Staff
  • Posts: 8916
  • Joined: Feb 02, 2011
|
#40724
Complete Question Explanation
(The complete setup for this game can be found here: lsat/viewtopic.php?t=1626)

The correct answer choice is (A)

This question is identical to Question #2 in both the question stem and the nature of the correct answer. As in #2, because there is only one available space in the appellate group, no two randoms can appear in the appellate group. Thus, answer choice (A)—J and M—can never occur, and is therefore the correct answer.

Get the most out of your LSAT Prep Plus subscription.

Analyze and track your performance with our Testing and Analytics Package.